inequalities
by Ducksohappi, May 16, 2025, 12:57 PM
let a,b,c be non-negative numbers such that ab+bc+ca>0. Prove:

P/s: I have analysed:
, similarly to
, by SOS

P/s: I have analysed:


This post has been edited 1 time. Last edited by Ducksohappi, 5 hours ago
A little problem
by TNKT, May 15, 2025, 1:17 PM
Problem. Let a,b,c be three positive real numbers with a+b+c=3. Prove that \color{blue}{\frac{1}{4a^{2}+9}+\frac{1}{4b^{2}+9}+\frac{1}{4c^{2}+9}\le \frac{3}{abc+12}.}
When does equality hold?
P/s: Could someone please convert it to latex help me? Thank you!
See also MSE: https://math.stackexchange.com/questions/5065499/prove-that-frac14a29-frac14b29-frac14c29-le-frac3
When does equality hold?
P/s: Could someone please convert it to latex help me? Thank you!
See also MSE: https://math.stackexchange.com/questions/5065499/prove-that-frac14a29-frac14b29-frac14c29-le-frac3
This post has been edited 1 time. Last edited by TNKT, Today at 1:23 AM
Trigonometric Product
by Henryfamz, May 13, 2025, 4:52 PM
Gcd(m,n) and Lcm(m,n)&F.E.
by Jackson0423, May 13, 2025, 4:12 PM
Find all functions
such that for all positive integers
,
where
and
denote the least common multiple and the greatest common divisor of
and
, respectively.


![\[
f(mn) = \mathrm{lcm}(m, n) \cdot \gcd(f(m), f(n)),
\]](http://latex.artofproblemsolving.com/6/0/7/6071692e2716901e88cc1cfc19ed60388c10701d.png)




IMO Genre Predictions
by ohiorizzler1434, May 3, 2025, 6:51 AM
Everybody, with IMO upcoming, what are you predictions for the problem genres?
Personally I predict: predict
Personally I predict: predict
ANG GCA
D1024 : Can you do that?
by Dattier, Apr 29, 2025, 5:11 PM
Let
and
.
Can you calculate
?


Can you calculate

This post has been edited 2 times. Last edited by Dattier, May 8, 2025, 8:14 PM
f(x + f(y)) is equal to x + f(y) or f(f(x)) + y
by parmenides51, Jul 20, 2024, 11:49 PM
Find all functions
satisfying the following condition: for any real numbers
and
, the number
is equal to
or 






Two lengths are equal
by 62861, Jul 7, 2016, 11:09 PM
Let
be a triangle with
. Let
,
, and
be the midpoints of the sides
,
, and
respectively. A circle
passing through
and tangent to
at
meets the segments
and
at
and
, respectively. The points
and
are symmetric to
and
about
and
, respectively. The line
meets
and
at
and
, respectively. The line
meets
again at
. Prove that
.
Proposed by El Salvador































Proposed by El Salvador
This post has been edited 1 time. Last edited by 62861, Jul 20, 2016, 7:44 AM
Reason: added proposer
Reason: added proposer
The sequence does not contain numbers of the form 2^n - 1
by Amir Hossein, Sep 2, 2010, 1:04 AM
Prove that the sequence
contains no term of the form 


The oldest, shortest words — "yes" and "no" — are those which require the most thought.
Archives














Shouts
Submit
118 shouts
Contributors
adityaguharoy • Akatsuki1010 • Amir Hossein • AndrewTom • arqady • CeuAzul • chocopuff • CJA • derangements • dgrozev • Grotex • Hypernova • j___d • Lonesan • Math_CYCR • pco • phi1.6180339.. • Pirkuliyev Rovsen • sqing • szl6208 • Tintarn • Virgil Nicula • xzlbq • Αρχιμήδης 6
Tags
About Owner
- Posts: 4657
- Joined: Apr 29, 2014
Blog Stats
- Blog created: Apr 26, 2016
- Total entries: 101
- Total visits: 27026
- Total comments: 61
Search Blog